Question:
Answer: The best answer is option A. This child’s presentation of palpable purpura, arthritis and arthralgias, abdominal pain and renal disease (hematuria) is classic for IgA vasculitis, an immune-mediated vasculitis caused by deposition of IgA immune complexes in multiple tissues, also known as Henoch-Schönlein purpura. It is primarily a disease of children, and often follows a recent upper respiratory infection, especially group A streptococcal infection. It is usually a benign, self-limited disease that does not require any treatment beyond supportive care.
Mixed cryoglobulinemia syndrome (B) is a type of immune vasculitis due to polyclonal IgG and IgM antibodies that causes a triad of purpura, arthralgia, and weakness due to peripheral neuropathy (“Meltzer’s triad”). Hepatic dysfunction is also quite common, and 20-30% of patients develop glomerulonephritis. Mixed cryoglobulinemia is most commonly seen in patients with hepatitis C infection.
Post-streptococcal glomerulonephritis (C) is caused by recent group A strep infection, and typically occurs in children. In addition to hematuria (which often causes cola-colored or tea-colored urine), symptomatic individuals will often have edema and hypertension. It does not cause arthralgias, abdominal pain, or purpura.
Rheumatic fever (D) is another complication of group A strep infection, usually occurring in children. It causes arthritis as well as a rash called erythema marginatum, a well-defined ring-like erythematous rash that begins on the trunk and gradually spreads outward before resolving spontaneously. Other prominent features of rheumatic fever include pancarditis, chorea, and painless subcutaneous nodules.
Scarlet fever (E) is a diffuse erythematous rash that may develop in association with group A strep pharyngitis. It begins as a rough, “sandpapery” rash in the axillae and groin, then progresses to involve the trunk as well as other flexural areas. It may also be associated with fever and a strawberry tongue.
Question:
Answer: The best answer is (A). This patient has symptoms of claudication (exertional pain in the legs, thighs or buttocks) and a physical examination suggestive of peripheral artery disease (PAD). His risk factors include his age, history of tobacco use and known atherosclerotic cardiovascular disease. Erectile dysfunction is commonly seen in men with significant aortoiliac atherosclerosis. The triad of claudication, diminished femoral pulses and erectile dysfunction is known as Leriche syndrome.
An ankle-brachial index (option A) is a relatively simple test to confirm the diagnosis of PAD. It is measured by calculating the ratio between the systolic blood pressure (SBP) at the ankle and the SBP in the brachial artery. An ankle-brachial index (ABI) ≥0.9 is normal, while an ABI <0.9 suggests the presence of PAD.
Beta blockers such as metoprolol (option C) are sometimes thought to cause erectile dysfunction, but some studies have suggested that this occurs in as little 0.5% of men taking beta blockers. Other studies have suggested that erectile dysfunction is more commonly seen in patients taking thiazide diuretics than beta blockers.
Question:
Answer: The best answer is option D. This patient’s presentation is consistent with hypercortisolism (Cushing syndrome), which causes hyperglycemia, moon facies, truncal obesity and purple abdominal striae. The most common cause of Cushing syndrome is exogenous glucocorticoid use. The second most common cause is thought to be ectopic production of adrenocorticotropic hormone (ACTH) from a non-pituitary site, most notably small cell lung cancer (option D). Lung cancer might also account for her reported weight loss.
Neither adenocarcinoma of the lung (option A) nor squamous cell carcinoma of the lung (option E) is commonly associated with ectopic ACTH production. Pituitary adenomas may produce excessive amounts of ACTH which leads to hypercortisolism (a condition called Cushing disease). ACTH-producing adenomas are called corticotroph adenomas and are fairly uncommon. The most common type of functioning pituitary adenoma is a lactotroph adenoma (option A), which produces prolactin, not ACTH. Hyperprolactinemia causes hypogonadism and possibly gynecomastia.
Adenomas that arise in the adrenal cortex may produce ACTH and cause Cushing syndrome. A pheochromocytoma (option C), however, is a neuroendocrine tumor that usually arises in the adrenal medulla. Pheochromocytomas produce catecholamines such as epinephrine and norepinephrine, leading to episodic hypertension, tachycardia, diaphoresis, and headache.
Question:
Answer: The correct answer is D. This patient’s death was most likely due to primary amebic meningoencephalitis (PAM), caused by infection with Naegleria fowleri, a free-living ameba found in warm freshwater sources. N. fowleri penetrates the nasal mucosa and then travels along the olfactory nerves to gain access to the brain. The resulting CNS infection is fatal in nearly all cases, with a mortality rate of 99%.
Foodborne illnesses (option A) tend to cause GI symptoms. One exception that does affect the CNS is variant Creutzfeldt-Jakob disease (vCJD), which is acquired by eating contaminated beef products. However, examination of the brain tissue in a patient with vCJD would show abnormally folded prion proteins, not trophozoites.
Contaminated water (option B) is the source of a number of infections, including those due to Giardia lamblia, Vibrio cholerae, and hepatitis A virus. However, these infections tend to cause GI, rather than CNS, symptoms.
Mosquitoes (option C) are vectors for many diseases, including some that affect the CNS, such as West Nile virus and St. Louis encephalitis. However, these are due to viral infections and would not show trophozoites on examination of the brain.
Hookworms (e.g., Necator, Ancylostoma) are found in the soil; they most commonly enter the body by penetrating through the skin of the feet (option E). Hookworm infections cause GI symptoms, nutritional deficiencies, and anemia.
Question:
Answer: The best answer is (B). This patient has hip osteomyelitis and resultant sepsis caused by methicillin-resistant Staphylococcus aureus (MRSA). His IV drug use is a significant risk factor for bloodstream and bone/joint infections. Osteomyelitis often presents with signs and symptoms of infection such as fevers and chills. The affected bone/joint is frequently erythematous, swollen and tender with decreased range of motion and/or weight-bearing ability. Laboratory markers of inflammation such as leukocyte count, ESR and CRP are frequently elevated. Diagnosis consists of imaging studies such as MRI, bone biopsy, and blood cultures. Empiric treatment should cover MRSA until blood and/or bone biopsy culture results are available. In this case the cultures grow MRSA, and vancomycin is considered first-line treatment. Vancomycin works by inhibiting bacterial cell wall biosynthesis by binding to D-alanine-D-alanine precursor thereby blocking peptidoglycan polymerization.
Option A is the mechanism of action of fluoroquinolone antibiotics, which are not indicated for treatment of MRSA infections since many strains of MRSA have developed resistance to fluoroquinolones.
Option C is the mechanism of action of linezolid, which is effective against MRSA but is not considered first-line therapy. Of note, macrolide antibiotics such as erythromycin also work by binding to 50S ribosomal subunits, but most strains of MRSA are resistant to macrolides.
Option D is the mechanism of action of aminoglycoside antibiotics such as gentamicin, and also tetracycline antibiotics. While both classes may have some activity against MRSA, neither is considered appropriate therapy for serious MRSA infections. Of note, the tetracycline antibiotic tigecycline is active against MRSA, but it has been associated with increased mortality and is only used when other effective antimicrobials are not available.
Option E is the mechanism of action of aztreonam, a monobactam antibiotic commonly used to treat serious gram-negative infections. Like other β-lactam antibiotics, aztreonam is not effective against MRSA.
Question:
Answer: The best answer is (B). Normally, the jugular venous pressure decreases during inspiration, due to decreased intrathoracic pressure and increased filling of the right atrium. Jugular venous distention during inspiration is called Kussmaul’s sign. (This is not to be confused with Kussmaul respirations, the deep, labored breathing seen in diabetic ketoacidosis.) Kussmaul’s sign is most closely associated with chronic constrictive pericarditis and may also be found in patients with right ventricular infarction, restrictive cardiomyopathy, and massive pulmonary embolism.
Kehr’s sign (option A) is referred pain in the left shoulder from splenic rupture.
Pulsus paradoxus (option C) is a >10 mm Hg decrease in the systolic blood pressure during inspiration. It is most commonly found in cardiac tamponade, and is not commonly found in constrictive pericarditis. (Incidentally, this finding was so named by the same Dr. Adolph Kussmaul.)
Pulsus parvus et tardus (option D) is a weak, delayed arterial pulse found in patients with valvular aortic stenosis.
Quincke’s sign (option E) is pulsation in capillaries of the nail beds, visible when pressure is applied to the fingertip. Although this “sign” is historically associated with valvular aortic regurgitation, it can be seen in normal subjects as well, and is thus considered useless by some authors.
Question:
Answer: The best answer is breach of duty (option A). Malpractice claims require 4 basic elements:
- Duty of care – The health care provider must have owed the patient a legal duty by being involved in the patient’s care.
- Breach of duty – The health care provider must have failed to perform that duty by failing to conform to the standard of care.
- Harm – The patient must have suffered some injury or harm.
- Causation – The breach of duty must have caused the harm.
“Intent” (option E) is not required for a malpractice claim.
In the case described, the patient clearly suffered damage (option B) and injury (option D). The patient was under the direct care of the hospitalist, who clearly owed him a duty of care (option C). But the vignette presents no evidence that the hospitalist breached his duty to the patient. The hospitalist followed proper sterile technique when inserting the central line. Appropriate therapy for VRSA was administered. For a valid malpractice claim, the plaintiff would need to demonstrate that the hospitalist was negligent in his delivery of care, and that this breach of duty caused the patient’s infection and subsequent death.
Question:
Answer: The best answer is option E. When platelets are activated they secrete numerous prothrombotic substances including thromboxane A2, fibrinogen, and adenosine diphosphate (ADP). The surfaces of platelets are covered with P2Y12 receptors and also with glycoprotein IIb/IIIa (GPIIb/IIIa), which is capable of binding to fibrinogen. The secreted ADP binds to the P2Y12 receptor, which leads to a downstream conformational change in the GPIIb/IIIa molecule that converts it from a low-affinity receptor to a high-affinity receptor for fibrinogen. As a result, GPIIb/IIIa on the surface of platelets binds to circulating fibrinogen. This “crosslinking” of multiple platelets with fibrinogen causes the platelets to aggregate and form a platelet plug, which is the first step in hemostasis. Clopidogrel acts early in this process by inhibiting the P2Y12 receptors and preventing platelet aggregation.
Option A describes the mechanism of action of the direct thrombin inhibitors such as argatroban and dabigatran, and also the mechanism of action of heparin, which activates antithrombin and promotes the inactivation of both thrombin and factor Xa.
Option B describes the mechanism of action of direct factor Xa inhibitors such as rivaroxaban, the indirect Xa inhibitor fondaparinux, and heparin.
Option C describes the mechanism of action of NSAIDs (which inhibit COX-1 and COX-2) and also the selective COX-2 inhibitors such as celecoxib.
Option D describes the mechanism of action of GPIIb/IIIa inhibitors such as abciximab, eptifibatide tirofiban. These drugs bind directly to GPIIb/IIIa on the platelet surface and prevent the crosslinking of platelets with fibrinogen. GPIIb/IIIa inhibitors are sometimes used following percutaneous coronary intervention, and also in some patients with non-ST elevation MI (NSTEMI).
Question:
Answer: The best answer is Salmonella enterica (option D). The blood smear shows evidence of sickle cell disease (SCD), which increases the risk of osteomyelitis. Sickling of red cells causes recurrent splenic infarction and autosplenectomy at an early age, rendering these patients susceptible to infections by encapsulated organisms such as S. pneumoniae, H. influenzae, N. meningitidis and Salmonella. Salmonella is the most common cause of osteomyelitis in SCD patients in the US and Europe. E. coli (option A) is another common cause of osteomyelitis in this population, but is considerably less common than Salmonella. Haemophilus influenzae (option B) is an uncommon cause of osteomyelitis, due to the effectiveness of the Hib vaccine. Neisseria meningitidis (option C) does not commonly cause osteomyelitis. Staphylococcus aureus (option E) accounts for one-half to two-thirds of cases of osteomyelitis in individuals without SCD in the US and Europe, but only one-fourth of cases of osteomyelitis in individuals with SCD. In sub-Saharan Africa and the Middle East, however, S. aureus remains the most common bacterial cause of osteomyelitis.
- George A, DeBaun, MR. Bone and joint complications in sickle cell disease. (Accessed on July 29, 2015.)
- Krogstad, P. Hematogenous osteomyelitis in children: Epidemiology, pathogenesis, and microbiology. In: UpToDate, Post TW (Ed), UpToDate, Waltham, MA. (Accessed on July 29, 2015.)
Question:
Answer: The best answer is (A). This patient has developed septic shock secondary to pneumonia. There is no evidence of CHF or cardiogenic shock on her physical examination. Septic shock is circulatory collapse resulting from peripheral vasodilation that comes about in response to infection.
- The mean arterial pressure (MAP) is low in all types of shock.
- Since septic shock results from peripheral vasodilation, the systemic vascular resistance (SVR) is low. SVR is also low in anaphylactic shock and neurogenic shock, both of which result from peripheral vasodilation.
- Pulmonary capillary wedge pressure (PCWP) is an indirect measure of pulmonary venous pressure and a good estimate of left atrial pressure. PCWP is normal or low in septic shock, but is generally elevated in cardiogenic shock, in which blood in the left ventricle backs up into the left atrium.
Question:
Answer: The best answer is chloramphenicol (option A). This patient has Rocky Mountain spotted fever (RMSF), which is caused by the bacterium Rickettsia rickettsii and transmitted by a tick vector. The illness begins 2-14 days following a tick bite, and may initially include fever, headache, myalgias, arthralgias, nausea, and abdominal pain. After 3-5 days, the macular rash develops on the wrists and ankles, eventually spreading to the trunk as described. If untreated, RMSF can cause fulminant organ failure and death. Laboratory studies are often unremarkable, although thrombocytopenia may eventually develop as the illness progresses, and CSF protein may be moderately elevated. Because R. rickettsii is an intracellular organism, it does not grow on routine blood cultures.
The preferred treatment for RMSF is doxycycline (option C), but tetracycline antibiotics are teratogenic and should be avoided in pregnant patients. Chloramphenicol (option A) is the recommended therapy for treatment of RMSF during pregnancy. Concerns about so-called “gray baby syndrome” due to chloramphenicol are likely unwarranted in this patient. Gray baby syndrome may rarely be seen when chloramphenicol is given to premature infants and neonates, not when it is given to pregnant women. Chloramphenicol given during pregnancy has not been associated with an increased risk of teratogenesis. Caution is recommended when using chloramphenicol in a pregnant patient near term or during labor, but the CDC affirms that chloramphenicol may be used for the treatment of RMSF in pregnant women.
Question:
Answer: The correct answer is (C). This patient’s uniocular central vision loss and eye pain suggests optic neuritis, which is confirmed by the MRI. Optic neuritis is a common presentation of multiple sclerosis (MS). Although the history given is insufficient to make a formal diagnosis of MS, the presence of oligoclonal bands supports the diagnosis.
Multiple sclerosis and optic neuritis are characterized by demyelinated plaques in the CNS, thought to be caused by autoimmune inflammation of the oligodendrocytes (option C) and eventually neuronal axons. Astrocytes (option A) and microglia (option B) are also CNS glial cells, like oligodendrocytes, but they do not form myelin and are not targeted in MS. Schwann cells (option E) form myelin in the peripheral nervous system, not the CNS. Photoreceptor cells (option D) such as rods and cones are found in the retina but not in the optic nerve, and are not affected in optic neuritis.
Question:
Answer: The best answer is D. Infarction of the lateral medulla (Wallenberg syndrome) is one of the most common posterior circulation stroke syndromes. The findings can include vertigo, nystagmus, hoarse voice, dysphagia, loss of pain and temperature sensation in the contralateral trunk and limbs, loss of pain and temperature sensation in the ipsilateral face, limb ataxia, and sometimes ipsilateral Horner syndrome.
Since this patient’s lesion is located in the left lateral medulla, disruption of the left spinothalamic tract would cause loss of pain and temperature sensation in the contralateral arm (option D), since the fibers carrying this sensory information decussate in the anterior white commissure of the spinal cord, before ascending to the brainstem.
Loss of pain and temperature sensation in the face is due to a lesion of the spinal trigeminal nucleus (which extends from the pons into the medulla). These fibers, however, do not decussate, so in a stroke of the lateral medulla the ipsilateral face is affected. (One hallmark of brainstem strokes is the presence of long tract symptoms on one side and cranial nerve symptoms on the other side.) Loss of pain and temperature sensation on the right face (option E) would suggest a stroke of the right lateral medulla.
Similarly, limb ataxia and past-pointing (option F) occur ipsilateral to the lateral medulla lesion, which can affect the inferior cerebellar peduncle.
Spastic hemiparesis of the contralateral limbs (option G) is seen in strokes of the medial medulla (not the lateral medulla) because of damage to the corticospinal tract, which is located in the medullary pyramids and decussates in the medulla.
Proprioceptive information (option C) travels from the limbs to the brain in the posterior columns of the spinal cord before decussating in the medulla and then continuing up the brainstem as the medial lemniscus. Since the medial lemniscus is located in the medial medulla, it would not be affected by a lateral medullary stroke.
Deviation of the tongue (option B) would likewise be seen in a stroke of the medial medulla, which is the location of the hypoglossal nucleus (CN XII). A stroke of the left hypoglossal nucleus would cause weakness of the left tongue muscles, so that attempts to protrude the tongue would cause it to deviate leftward (toward the side of the stroke).
Facial droop (option A) could be caused by either a stroke of the cortex or a stroke of the facial nerve nucleus, which is located not in the medulla but in the pons (specifically the lateral inferior pons).
Question:
Answer: The best answer is degmacytes (option B). This patient has hemolytic anemia following ingestion of fava beans, which strongly suggests a diagnosis of glucose-6-phosphate dehydrogenase (G6PD) deficiency. G6PD is an enzyme that is required to regenerate NADPH in order to neutralize reactive oxygen species. G6PD-deficient individuals develop hemolytic anemia after ingesting drugs and foods (such as fava beans) that induce oxidative damage.
The presence of Heinz bodies or degmacytes in a patient with hemolytic anemia is highly suggestive of G6PD deficiency. Heinz bodies are clumps of denatured hemoglobin that accumulate in these individuals’ RBCs due to oxidative damage. Degmacytes are RBCs with a small “bite” taken out of them. It has been hypothesized that degmacytes are formed when splenic macrophages remove the Heinz bodies (and part of the RBC’s cytoplasm) from the damaged RBCs. Another hypothesis is that the bite-like deformities are hemoglobin-free areas of the RBC that do not stain.
Basophilic stippling (option A) describes the presence of numerous basophilic granules within the RBC cytoplasm, due to precipitation of ribosomes and ribosomal RNA. Basophilic stippling can be seen in lead poisoning, thalassemia, anemia of chronic disease, and alcohol abuse.
A Howell-Jolly body (option C) is a basophilic remnant of a nucleus within a RBC. Howell-Jolly bodies occur in patients with asplenia, since the spleen usually removes these cells from circulation.
Reticulocytes (option D) are immature RBCs which are normally found in circulation in very small amounts. Many types of hemolytic anemia result in increased reticulocytes in the peripheral blood; this finding is nonspecific.
Spherocytes (option E) are spherical RBCs that are typically smaller than normal RBCs and are identified by the absence of the characteristic central pallor. Spherocytes are nonspecific and may be seen in many types of hemolytic anemia as well as in hereditary spherocytosis.
Question:
Answer: The best answer is insulin (option D). In addition to facilitating the uptake of glucose in skeletal muscle and adipose tissue, insulin promotes the cellular uptake of potassium. Intravenous administration of insulin begins to lower serum potassium levels within 10-20 minutes. The effect peaks at about 1 hour, and lasts 4-6 hours.
The most important first step in treating hyperkalemia with ECG changes is IV calcium gluconate (option B) or IV calcium carbonate. This treatment, however, is not intended to lower serum potassium levels. IV calcium stabilizes myocardial cell membranes and protects against life-threatening arrhythmias such as AV block, ventricular fibrillation, and asystole.
Like insulin, β agonists like albuterol (option A) will also shift potassium into the cells and lower serum potassium, but this takes about 90 minutes to start working. Sodium bicarbonate (option E) induces cells to secrete H+ to maintain the normal serum pH; and serum potassium declines as the cells concomitantly take up potassium in order to maintain electroneutrality. This process takes at least 1-2 hours, and may only be beneficial in patients with underlying metabolic acidosis.
Loop diuretics like furosemide (option C) lower serum potassium by promoting renal excretion of potassium. This is a longer-term therapy that is not appropriate for emergency short-term management of hyperkalemia.
Question:
Answer: The best answer is (C). This patient has Whipple’s disease, caused by Tropheryma whipplei infection of the intestine and other tissues. The disease predominantly affects middle-aged Caucasian men and causes malabsorption, weight loss, diarrhea, and abdominal pain. 65-70% of patients present with migratory arthralgias and polyarthritis which begin several years before the gastrointestinal symptoms. Neurological manifestations may include cognitive dysfunction, cerebellar ataxia, and disorders of the extraocular muscles.
Whipple’s disease is diagnosed primarily by endoscopic small bowel biopsy showing shows villous atrophy and so-called “foamy” macrophages in the lamina propria. These macrophages are filled with T. whipplei, which stain bright pink with periodic acid-Schiff stain (option C).
Question:
Answer: The correct answer is (A). Amyloid beta (Aβ) is formed by the proteolytic cleavage of amyloid precursor protein (APP), which is encoded by a gene found on chromosome 21. Mutations in the APP gene increase the risk of early-onset Alzheimer disease. Because individuals with trisomy 21 have an extra copy of this chromosome and this gene, there is a high prevalence of Alzheimer disease in this population.
- The gene for Apolipoprotein E (option B) is found on chromosome 19.
- The gene for presenilin 1 (option C) is found on chromosome 14.
- The gene for presenilin 2 (option D) is found on chromosome 1.
- The gene for tau protein (option E) is found on chromosome 17.
Question:
Answer: The best answer is (E) quinidine. This patient presents with symptoms of cinchonism, a syndrome caused by the ingestion of derivatives of cinchona bark such as quinine and quinidine. The key features of cinchonism include tinnitus (and often hearing loss), blurred vision, headache, vomiting, and diarrhea. Confusion, delirium and psychosis may occur. Quinidine toxicity will also cause bradycardia and hypotension, and a lichenoid photosensitivity reaction is possible.
Salicylates such as aspirin (option B) can cause a cinchonism-like presentation of tinnitus and vomiting, but does not typically cause headaches, vision disturbances, or rash. Aspirin also stimulates the medullary respiratory center and causes tachypnea (rapid breathing) and hyperventilation (deep breathing) which are not present in this patient. Aspirin toxicity also typically causes sinus tachycardia rather than bradycardia.
Amiodarone (option A) is a well-known cause of photosensitivity, and it can cause optic neuropathy that may lead to permanent blindness. It does not cause tinnitus, however, and it usually only causes hypotension wen administered intravenously.
Digoxin toxicity (option C) can cause many of this patient’s findings, including bradycardia, hypotension, vision disturbances, and headache, but digoxin is not associated with tinnitus. Phenytoin (option D) can cause blurred vision, headache, vomiting, and various dermatological eruptions, but it does not cause tinnitus, and findings of bradycardia and hypotension are rare (and more commonly associated with intravenous administration).
Question:
Answer: The best answer is pericardiocentesis (option D). This patient demonstrates Beck’s triad of cardiac tamponade, which consists of hypotension, jugular venous distension and muffled heart sounds. Other signs of cardiac tamponade can include tachycardia (which is present in this patient) and pulsus paradoxus (which is not described). The diagnosis of tamponade might be confirmed by echocardiography, but because this patient is in cardiogenic shock the most appropriate intervention is to perform pericardiocentesis to remove the blood that has accumulated within the pericardial space.
IV fluids (option A) might be appropriate in a patient with hypovolemic shock, such as from acute blood loss. The distended neck veins, however, suggest cardiogenic shock rather than hypovolemia. Morphine (option B) and nitroglycerin (option E) might be appropriate treatments for myocardial infarction, but either treatment would need to be used with extreme caution in a patient with such significant hypotension. Emergency needle thoracostomy (option C) would be appropriate management for a suspected tension pneumothorax, which could also present with dyspnea, chest pain and distended neck veins. The fact that this patient has normal bilateral breath sounds argues strongly against a diagnosis of tension pneumothorax.
This patient has a cluster headache, which is a severe unilateral headache that recurs at the same time and location daily for several weeks. The headache is usually located around one eye and is accompanied by autonomic symptoms (e.g., ptosis, miosis, conjunctival injection, lacrimation, diaphoresis, nasal congestion and rhinorrhea). Cluster headaches are most common in young, male smokers.
Cluster headaches can be aborted by administering 100% oxygen (option A) by non-rebreather mask for 15-20 minutes. Headaches that do not respond to oxygen may be treated with a subcutaneous dose of a serotonin agonist such as sumatriptan (option C), but the risk of side effects makes 100% oxygen a better first-line choice. Serotonin agonists are more commonly used to treat migraine headaches.
High-dose glucocorticoids such as prednisone (option B) and checking the ESR (option D) would be appropriate for a patient with suspected giant cell arteritis (also known as temporal arteritis). Giant cell arteritis is most commonly seen in elderly women and generally presents with a unilateral headache not accompanied by sympathetic symptoms.
CT scan of the head (option E) is not indicated for cluster headache.
Question:
Answer: The correct answer is (B). The finding described is Trousseau’s sign of latent tetany, in which occlusion of the brachial artery for 3-5 minutes leads to carpopedal spasm. Trousseau’s sign is associated with hypocalcemia, which characteristically causes prolongation of the QT interval on ECG (option B). A prolonged QT interval may also be seen in patients with hypermagnesemia.
J waves (option A) are a finding seen in hypothermia. Prominent U waves (option C) maybe seen in hypokalemia. Tall, peaked T waves (option D) are classically seen in hyperkalemia, and may sometimes be seen in hypomagnesemia. Widening of the QRS complex (option E) may be seen in hyper- or hypokalemia and hyper- or hypomagnesemia.
Question:
Answer: The best answer is (E). This patient’s presentation suggests syringomyelia, in which a longitudinal cavitary lesion develops in the spinal cord as a late sequela of spinal cord injury or a Chiari malformation. Syringomyelia is most commonly found in the cervical cord and therefore affects the upper extremities. As the syrinx cavity enlarges it disrupts the fibers of the spinothalamic tract in the anterior white commissure, leading to loss of pain and temperature sensation in the upper extremities. This is often described as a “cape-like” distribution of sensory loss over the shoulders, arms and hands. Further expansion of the syrinx may damage the nearby anterior horns, resulting in lower motor neuron findings (e.g., weakness, muscle wasting, fasciculations and areflexia). The treatment for syringomyelia is surgical decompression of the cavity (option E).
Atorvastatin and clopidogrel (option A) would be appropriate therapy for an ischemic stroke, which would cause upper motor neuron signs (e.g., spastic paralysis and hyperreflexia). Sensory deficits would most likely involve both the spinothalamic tract (e.g., pain and temperature sensation) and the dorsal columns (e.g., vibration sense, fine touch and proprioception).
Interferon beta (option B) is used to treat multiple sclerosis (MS), which can certainly cause spinal cord lesions, but usually also causes demyelinating lesions in the brain. Another hallmark of MS is that lesions occur in multiple locations within the CNS, and typically MS follows a relapsing-remitting course, where new lesions (and symptoms) appear and spontaneously resolve over time.
Plasmapheresis (option C) might be an excellent treatment for a patient with Guillain-Barré syndrome (GBS) or myasthenia gravis (MG). GBS causes muscle weakness that typically begins in the legs and ascends to involve the arms, the face and oropharynx, and sometimes the respiratory muscles. Sensory deficits are uncommon. Likewise, MG is caused by autoantibodies against the acetylcholine receptors at the neuromuscular junction, which leads to fluctuating muscle weakness without sensory deficits.
Riluzole (option D) is a drug used to treat amyotrophic lateral sclerosis, a disease of both upper and lower motor neurons that typically causes very little sensory disturbance.
Question:
Answer: The best answer is conversion disorder (option C). Conversion disorder is characterized by the presence of neurological symptoms that are incompatible with a recognized neurological or medical condition. This patient’s episodes are clearly nonepileptic in nature. Conversion disorder may or may not be associated with a specific psychological stressor, such as this patient’s witnessing the death of her boyfriend.
Acute stress disorder (option A) and posttraumatic stress disorder (option E) are closely tied to specific stressful or traumatic events, but the symptoms include intrusive memories of the traumatic event, avoidance of stimuli associated with the event, negative alterations in cognition or mood, and alterations in arousal.
Adjustment disorders (option B) are characterized by the development of behavioral or emotional symptoms (such as depressed mood, anxiety, or disturbance of conduct) in response to a specific stressor.
Factitious disorder (option D) is the falsification of physical or psychological signs or symptoms, or induction of injury or disease, with a clear intent to deceive (even in the absence of obvious external rewards). There is nothing in the given clinical scenario that indicates willful deception or deliberate falsification of symptoms.
Question:
Answer: The correct answer is (D). The nodules depicted are Kimmelstiel-Wilson nodules, seen in diabetic nephropathy. Hemoglobin A1c is elevated in patients with diabetes mellitus.
Anti-dsDNA antibodies (option A) are commonly present in patients with lupus nephritis. ANCA (option B) can be associated with a variety of diseases that cause glomerulonephritis, such as granulomatosis with polyangiitis (Wegener’s) and eosinophilic granulomatosis with polyangiitis (Churg-Strauss). Antistreptolysin O antibodies (option C) are found in individuals with recent group A streptococcal infection, and would thus be more likely to be positive in a patient with poststreptococcal glomerulonephritis. Hepatitis B infection (option E) is associated with both membranous nephropathy and membranoproliferative glomerulonephritis (MPGN).
Question:
Answer: The correct answer is (C). This patient’s presentation strongly suggests “periodic fever, aphthous stomatitis, pharyngitis, and adenopathy (PFAPA) syndrome,” which is a pediatric illness characterized by:
- Recurrent fevers occurring every 28 days and lasting 3-6 days
- Aphthous ulcers, which may precede the onset of the fever by a day
- Pharyngitis with tonsillar exudates
- Tender cervical lymphadenopathy
PFAPA syndrome generally does not cause coryza, cough, diarrhea, rash, or arthritis/arthralgias. Between the episodes of illness, the child is generally healthy and exhibits normal growth and development.
One characteristic of PFAPA syndrome is that the fever and pharyngitis respond very rapidly to a single low dose of a glucocorticoid such as prednisone (option C), often within a few hours. The episodic illness can occur for 5-8 years but generally resolves spontaneously and without sequelae.
Amoxicillin (option A) would be appropriate for a bacterial infection such as streptococcal pharyngitis, but this does not cause aphthous ulcers or a predictable, episodic fever. Aspirin (option B) is the treatment for Kawasaki disease, which causes fever and lymphadenopathy but also rash, conjunctivitis, cracked lips, and strawberry tongue. Valacyclovir (option D) would be appropriate for treating an oral herpes simplex virus infection, which can cause fever and pharyngitis, but oral herpes lesions are grouped vesicles on an inflamed, erythematous base. Vitamin A (option E) might be used to treat measles (rubeola) infection, which causes fever and oral lesions called Koplik spots and may cause pharyngitis and lymphadenopathy. But the prodrome of measles causes cough, coryza, and conjunctivitis followed by a prominent rash, none of which are present in this patient.
Question:
Answer: The best answer is option A. Listeria monocytogenes is a motile, gram-positive bacillus that may infect humans through contaminated food. Listeriosis can affect immunocompromised individuals and pregnant women, especially in the third trimester. Neonatal meningitis in the first week of life is most commonly due to transmission from mother to child during delivery. Group B streptococcus (option B) and gram-negative bacilli such as E. coli (option E) are the most common causes of neonatal meningitis in developed countries. Although Listeria accounts for a small percentage (< 5%) of all cases of neonatal meningitis, it is a classic pathogen in this age group. Likewise, coagulase-negative staphylococcus (option C) and Enterococcus species (option D) are infrequent causes of meningitis in neonates.
Question:
Answer: The correct answer is (A). Autism spectrum disorder (ASD) is a neurodevelopmental disorder that is generally characterized by:
1) Restricted or repetitive patterns of behavior and interest, and
2) Deficits in social interaction and communication
The DSM-5 diagnostic criteria include “deficits in developing, maintaining and understanding relationships,” which may include difficulty making friends (option A).
Some individuals with ASD do exhibit special “savant” skills (option E) in memory, mathematics, music, or art despite deficits in other areas, but this is not required for the diagnosis, and is not particularly common. Likewise, these individuals may or may not have intellectual disability.
ASD generally has onset in early childhood, often before 3 years of age. Individuals with less severe ASD may not be recognized until school age (option C), when the demands placed upon them by increased socialization exceed their capabilities.
Because children with ASD tend to be inwardly focused, infants and toddlers are often content with playing by themselves and not demanding of caregiver attention (option D). Hyperactive behavior (option B) is generally not seen in autistic spectrum disorder.
Question:
Answer: The best answer is (C). Non-ergot dopamine agonists such as ropinirole and pramipexole are generally the first-line treatment for patients with moderate or severe restless legs syndrome (RLS). RLS causes an uncomfortable urge to move the legs when at rest, especially late in the day. This discomfort is quickly relieved by walking or stretching the legs, but it usually recurs quickly once the legs are again at rest.
Benzodiazepines (option B) are sometimes used for patients with mild or intermittent RLS symptoms, but may lead to dependence. Anticonvulsants such as gabapentin and pregabalin may also be used.
Question:
Answer: The correct answer is (C). Carotid artery atherosclerosis can result in embolic occlusion of the central retinal artery, resulting in sudden onset of painless, monocular vision loss. The funduscopic examination shows a pale retina (due to ischemia) with a “cherry-red spot” at the macula. This is an ophthalmologic emergency, as it may result in irreversible ischemic damage and permanent vision loss.
Retinal detachment (option E) also causes sudden onset of painless, monocular vision loss, but the vision loss usually resembles a curtain coming down over the field of vision, and is usually accompanied by floaters and/or flashing lights. The entire visual field is generally not obscured, since the portion of the retina that remains attached is still functioning normally. The detached portion of the retina looks wrinkled or billowed on funduscopic examination.
Age-related macular degeneration (option B) most commonly causes slowly progressive, bilateral vision loss. The funduscopic examination typically shows small flecks of yellow extracellular debris called drusen.
Open-angle glaucoma (option D) likewise causes progressive, bilateral vision loss. Acute angle-closure glaucoma (option A) causes acute, monocular vision loss associated with eye pain, conjunctival redness, corneal edema, and a pupil that does not react to light.
Question:
Answer: The best answer is (E). As of 2008, the USPSTF recommends colon cancer screening using fecal occult blood testing (FOBT), sigmoidoscopy, or colonoscopy beginning at age 50 and continuing until age 75. The preferred frequency of screening varies with each screening modality: FOBT is performed annually, sigmoidoscopy is performed every 5 years, and colonoscopy is performed every 10 years.
The task force has found insufficient evidence to assess the benefits and harms of fecal DNA testing (option A) and CT colonography (option B), so neither of these modalities is recommended. The guidelines recommend against screening adults age 76 and older (option D).
A single stool specimen obtained during a rectal examination (option C) is not considered an adequate screen for colorectal cancer. FOBT should be performed by having the patient submit samples from three successive stools, which are then tested for the presence of blood using one of several methods.
NOTE: These recommendations may be altered by updated USPSTF guidelines scheduled for publication later in 2015.
Question:
Answer: The correct answer is (B). The question is asking for the proportion of all negative test results that are true negatives, which is called the negative predictive value. The data could be organized as follows:
The negative predictive value equals the number of true negatives (780) divided by the total number of negative results (780 + 220), which is 78%.
Question:
Answer: C. This patient has neuroleptic malignant syndrome (NMS), which is characterized by altered mental status, muscle rigidity (leading to hyperthermia and rhabdomyolysis), and autonomic instability (tachycardia and hypertension). The drugs associated with the highest risk of NMS are the high-potency traditional neuroleptics such as fluphenazine (option C). Atypical antipsychotics such as aripiprazole (option A) are less likely to cause NMS. Dantrolene (option B) is a drug that reduces skeletal muscle contraction by blocking the release of calcium from the sarcoplasmic reticulum. It is used to treat malignant hypertension and is sometimes used off-label to treat NMS.
Monoamine oxidase inhibitors such as phenelzine (option D) and selective serotonin reuptake inhibitors such as sertraline (option E) may be associated with an increased risk of serotonin syndrome, which is characterized by altered mental status, autonomic instability, hyperreflexia, clonus, and in severe cases may result in muscle rigidity and hyperthermia. Clinically, serotonin syndrome can be distinguished from NMS by the rapid onset (serotonin syndrome develops over 24 hours while NMS develops over days to weeks) and by the presence of hyperreflexia and clonus, which are characteristic of serotonin syndrome but not NMS.
Question:
Answer: B. This peripheral blood smear shows spherocytosis, in which the red blood cells (RBCs) become somewhat spherical, losing their normal biconcave shape and central pallor. MCHC is elevated because of the reduced RBC volume. Spherocytes are prone to hemolysis in the spleen, resulting in anemia, jaundice and splenomegaly. Spherocytosis may result from several disorders, including hereditary spherocytosis, autoimmune hemolytic anemia (AIHA), and glucose-6-phosphate dehydrogenase (G6PD) deficiency. AIHA is characterized by a positive direct antiglobulin test (Coombs test). Hemolytic episodes in G6PD deficiency are associated with Heinz bodies and degmacytes (bite cells) on the peripheral smear.
Hereditary spherocytosis will often result in a positive osmotic fragility test (option B), in which the patient’s RBCs readily lyse in a hypotonic solution. The Ham test (option A) is an older test for paroxysmal nocturnal hemoglobinuria, performed by measuring the propensity of RBCs to lyse in an acidic solution. Low ferritin (option C) is associated with iron deficiency anemia, which causes a hypochromic, microcytic anemia. Likewise, lead toxicity (option D) causes hypochromic, microcytic anemia, and the RBCs may show basophilic stippling. Vitamin B12 (option E) deficiency causes macrocytic anemia.
Question:
Answer: Vitamin A (option E). This patient is in the prodromal stages of measles. Measles infection has an incubation period of 8-10 days, followed by a prodromal illness that typically lasts 2-3 days. The prodrome begins with fever, anorexia and malaise, followed by cough, conjunctivitis and nasal mucosal inflammation (“coryza”). The exanthem seen on the oral mucosa is consistent with Koplik spots, which are pathognomonic for measles infection. Koplik spots usually precede the characteristic maculopapular rash of measles by about 2 days. Lymphadenopathy, which is usually absent during the prodrome, commonly develops with the exanthem.
Generally speaking, measles is treated with supportive measures such as antipyretics and fluids. In developing countries, treatment with vitamin A (option E) has been shown to reduce morbidity and mortality in measles patients. The World Health Organization (WHO) recommends administration of vitamin A to all children with measles in developing countries. There have been no clinical trials of vitamin A for measles in the United States.
There is no role for antibiotics such as amoxicillin (option C) in the treatment of viral infections such as measles. In vitro studies show susceptibility to the antiviral ribavirin, but this has not been studied in clinical trials. Acyclovir (option A) and amantadine (option B) are antivirals used for herpes virus and influenza virus, respectively. Aspirin (option D) is generally not given to children because of concerns about causing Reye syndrome.
Question:
Answer: Respiratory syncytial virus-D. This patient has mumps, which is characterized by a prodrome of fever, myalgias, headache, and anorexia, followed by painful swelling of the parotid glands (parotitis) that can last 7-10 days. Complications may include aseptic meningitis and orchitis (inflammation of the testes), with infertility as a potential sequela.
The mumps virus is a paramyxovirus with a negative-sense ssRNA genome. Of the viruses listed, only respiratory syncytial virus (option D) is a member of the Paramyxovirus family. Influenza virus (option C) is also a negative-sense ssRNA virus but is a member of the Orthomyxovirus family. Coronavirus (option A) is a positive-sense ssRNA virus that causes the common cold. Epstein-Barr virus (EBV) (option B) is a DNA virus and a member of the Herpesvirus family. EBV causes a variety of clinical illnesses including infectious mononucleosis, which presents with fever, fatigue, pharyngitis, tonsillitis, and cervical lymphadenopathy. Rubella virus (option E) is a positive-sense ssRNA virus of the Togavirus family. The combined measles-mumps-rubella (MMR) vaccine is given to reduce the number of injections needed, not because of structural similarity of the viruses.
Question:
Answer: D. This patient most likely has cold agglutinin autoimmune hemolytic anemia. Of the disorders listed, Mycoplasma pneumoniae infection (option D) is most closely associated with cold agglutinins. The patient has acrocyanosis of the fingers that develops following exposure to the cold and improves with warming. This should be distinguished from Raynaud phenomenon, which is characterized by sharply demarcated pallor followed by cyanosis and then erythema as perfusion is restored. The cold agglutinins associated with Mycoplasma infection may be found about two weeks after the onset of infection and usually resolve within 3-4 months.
Cold agglutinins may be associated with chronic lymphocytic leukemia but not typically with AML (option A). HIV infection (option C) and SLE (option E) are more likely to be associated with the production of warm agglutinins rather than cold agglutinins. SLE may also cause anemia of chronic disease, which is associated with a low (rather than high) reticulocyte count. DIC (option B) causes intravascular hemolysis but not acrocyanosis. DIC would typically also be associated with a low platelet count.
Question:
Answer: A) Fasting plasma glucose of 130 mg/dL.
Question:
Answer: B) treat compartment syndrome. Compartment syndrome occurs when an injury causes increased pressure within an enclosed compartment of an extremity, leading to impaired circulation. It classically causes pain, paresthesias, pallor, poikilothermia, diminished pulses and occasionally paralysis. Swelling of the affected extremity is common. The limb pain is typically worsened by external compression of the affected compartment or by passive stretching of the muscles within the compartment. The diagnosis of compartment syndrome can be confirmed by needle manometry, which demonstrates elevated pressure within the affected compartment(s).
Compartment syndrome is treated with fasciotomy, in which incisions are made in the bands of fascia to allow the intracompartmental pressures to normalize. Clindamycin (option A) might be appropriate treatment for cellulitis, although cellulitis typically causes redness and warmth of the affected limb, as well as fever. Heparin (option C) would be appropriate treatment for a deep venous thrombosis, which does not cause diminished arterial pulses or severe pain. Naproxen (option D) and rest, ice, compression and elevation (option E) would be appropriate for a sprain or other mild musculoskeletal injury but are not indicated for treatment of compartment syndrome.
Question:
Answer: Lung cancer (D). According to 2014 data released by the American Cancer Society, there are an estimated 310,010 cancer deaths among U.S. men annually. Nearly 87,000 of these deaths are caused by lung cancer, making it the #1 cause of cancer death among U.S. men. Prostate cancer (option G) is the most common cancer among men overall, with an estimated 233,000 cases diagnosed each year, but it is the second most common cause of cancer death, causing 29,480 deaths annually. Colon cancer (option A) kills an estimated 26,270 men annually, and pancreatic cancer (option F) kills 20,170. Of note, basal cell skin cancers (option B) are not reported in cancer registries, and while they are very common they rarely cause death.
Question:
Answer: B - E. coli. This patient’s presentation suggests acute bacterial prostatitis, which typically causes symptoms of urinary tract irritation (frequency, urgency, incontinence), dysuria, fever, and sometime perineal pain. Symptoms of urinary retention (e.g., hesitancy) are less common. Cloudy urine is possible, but penile discharge is uncommon. Examination of the prostate is notable for exquisite tenderness, and the prostate is often described as swollen, soft, and “boggy.”
Most cases of acute bacterial prostatitis are caused by Gram-negative bacteria, with E. coli being responsible for 60-90% of cases. Klebsiella (option C) and Enterobacteriaceae species account for 3-10% of cases. Proteus mirabilis is a rare cause of prostatitis, accounting for only 3-6% of cases. Chlamydia trachomatis (option A) and Neisseria gonorrhoeae (option D) may cause prostatitis in addition to urethritis and epididymitis, but these are usually seen in patients with STI risk factors and are classically suspected in younger patients.
Empiric treatment of acute bacterial prostatitis usually includes either trimethoprim-sulfamethoxazole or a fluoroquinolone (e.g., ciprofloxacin or levofloxacin), which penetrate the prostate well. Urine culture results should be used to guide further antimicrobial therapy. Antibiotics should be continued for 4-6 weeks.
Question:
Answer: E - Phenytoin. Excessive growth of terminal hair in a male-like pattern on a woman is called hirsutism. Is it most commonly due to hyperandrogenism. This patient, however, is described as having global abnormal hair growth, which suggests hypertrichosis rather than true hirsutism. Drugs that cause hypertrichosis include cyclosporine, minoxidil, penicillamine, and phenytoin (option E). Danazol (option A) is a drug that suppresses pituitary FSH and LH production and is used to treat endometriosis. It may cause true hirsutism but not hypertrichosis. Finasteride (option B) is an inhibitor of 5α-reductase, the enzyme that converts testosterone to the more potent androgen dihydrotestosterone. Finasteride is used to treat conditions in which high testosterone levels are problematic, including benign prostatic hyperplasia and male-pattern hair loss. It does not cause hypertrichosis. Flutamide (option C) is a drug that directly antagonizes testosterone receptors and is used to treat prostate cancer. Side effects may include decreased libido and sexual dysfunction but not increased hair growth. Ketoconazole (option E) is an antifungal with antiandrogen effects. It is known to cause gynecomastia but not hypertrichosis.
Question:
Answer: E - Epinephrine. This patient is potentially having an anaphylactic reaction to the candy (possibly to peanuts, a common allergen found in many candy bars). Anaphylaxis occurs when antigens bind to allergen-specific IgE on the surface of mast cells and basophils, resulting in the rapid release of mediators such as histamine, leukotrienes and cytokines. These mediators cause increased vascular permeability, vasodilation and bronchoconstriction, which may lead to cardiovascular collapse and respiratory failure. 90% of anaphylaxis patients will have cutaneous manifestations such as pruritus, flushing and generalized hives. Food allergies typically cause gastrointestinal symptoms such as vomiting, crampy abdominal pain, and eventually diarrhea.
Intramuscular epinephrine is the treatment of choice for anaphylactic reactions. Not only does epinephrine reduce mast cell degranulation, but it also reverses or prevents bronchoconstriction, upper airway edema and hemodynamic collapse.
Inhaled albuterol (option A) will reverse bronchoconstriction in anaphylaxis and is used as an adjunct to epinephrine. It will not, however, treat the oropharyngeal edema which may contribute to dyspnea in these patients.
H1 antihistamines such as cetirizine (option B) and diphenhydramine (option D) are adjuncts to epinephrine in treating anaphylaxis, primarily for treating pruritus and urticaria. Antihistamines do not relieve airway obstruction or hemodynamic collapse.
Question:
Answer: D. The mammographic feature that is most specific for breast cancer is the presence of a spiculated soft tissue mass. Named after a type of spear used by Roman soldiers, a spicule is a thin, needlelike projection on the surface of the mass. In almost 90% of cases, spiculated breast lesions indicate invasive breast cancer. Cysts, round or oval lesions, and lobulated masses are much less likely to be malignant.
Clustered particles of calcium (clustered microcalcifications) are often indicative of breast cancer. Roughly 60% of cancers identified on mammography contain clustered microcalcifications, although the absence of microcalcifications on mammography does not rule out breast cancer. Vascular calcifications (option E) are generally considered benign, as are smooth round calcifications, large coarse calcifications, and cutaneous calcifications.
Question:
Answer: A. Lobular carcinoma in situ is characterized by malignant cells arising within the breast lobules that do not penetrate the basement membrane and invade the breast stroma. LCIS cells typically show underexpression of the cell adhesion molecule E-cadherin (option A), which functions as a tumor suppressor. Lobular carcinomas are almost universally estrogen receptor-positive (option B), making treatment with selective estrogen receptor modulators (SERMs) such as tamoxifen particularly useful (option E). Most lobular carcinomas do not overexpress HER2/neu (option C). There are no particular mammographic findings suggestive of LCIS, and the microcalcifications seen in ductal carcinomas are not typically seen (option D).
Question:
Answer: This lesion is infiltrating lobular carcinoma (option B), the second most common type of invasive breast cancer in the US. (Infiltrating ductal carcinoma is by far the most common.) Infiltrating lobular carcinoma typically features single-file columns of cells arranged in concentric “target-like” sheets. Signet ring cells, in which the cytoplasm is filled with mucinous material that pushes the nucleus to the periphery, giving the appearance of a signet ring, can also be seen.Ductal carcinoma in situ (option A) is commonly found incidentally on biopsies performed for some other palpable lesion. DCIS involves proliferation of malignant cells within the breast ducts, without invasion through the basement membrane into the surrounding breast stroma.
Medullary carcinoma of the breast (option C) is a rare form of breast cancer. The tumor is well-circumscribed and is characterized microscopically by a prominent lymphocytic infiltrate.
Paget disease of the breast (option D) is a form of breast cancer that appears on the surface of the breast, on the nipple and areola. There may or may not be a concomitant palpable breast lesion. Histologically, adenocarcinoma cells are found within the epidermis of the nipple.
A phyllodes tumor (option E) is a bulky breast tumor similar to a sarcoma, usually characterized as having “leaf-life” papillary projections on histology. Most phyllodes tumors are benign, but they can undergo malignant transformation.
Question:
A 42-year-old man from Liberia is brought to an emergency department in Dallas with a 4-day history of fever, chills, malaise, headache, and myalgias. Which of the following cytokines is most likely involved in causing this patient’s fever?
(A) Fas ligand (FasL)
(B) Interferon alpha (IFN-α)
(C) Interleukin-6 (IL-6)
(D) Interleukin-10 (IL-10)
(E) Leukotriene B4
Answer: The best answer is interleukin-6 (option C). IL-6 is an important pro-inflammatory cytokine that mediates fever and acute inflammation, along with IL-1 and TNF-α. FasL (option A) is related to tumor necrosis factor and plays an important role in apoptosis. IFN-α (option B) is a cytokine produced by virus-infected cells that stimulates the destruction of infected cells. IL-10 (option D) is an antiinflammatory cytokine that inhibits the actions of macrophages and Th1 cells. Leukotriene B4 (option E) is a pro-inflammatory and chemoattractant cytokine that is best known for its role in neutrophil chemotaxis (along with IL-8 and C5a).
Question:
Answer: Nitroglycerin (option E). This patient has a history of dilated cardiomyopathy (most commonly due to ischemic heart disease), and he now presents with acute decompensated heart failure (ADHF) and pulmonary edema. Of the choices given, intravenous nitroglycerin would begin to relieve his symptoms most rapidly. A loop diuretic such as furosemide (option C) would also be indicated for acute treatment of CHF, but would likely take longer to provide relief than nitroglycerin. While in a healthy individual an IV dose of a furosemide could begin working in as little as 10-15 minutes, in a patient with low cardiac output and poor renal perfusion furosemide-induced diuresis could take up to 90 minutes to begin. Additionally, diuretics only benefit patients who are volume overloaded. Up to 50% of patients with cardiogenic pulmonary edema may be euvolemic or hypovolemic. Although the fluid is inappropriately distributed in the lungs, these patients are often intravascularly volume depleted. Nitrates such as nitroglycerin work by dilating peripheral veins, which promotes redistribution of fluid from the lungs to the periphery and provides more rapid relief of dyspnea.
Digoxin (option A) has been proven to improve symptoms and improve quality of life in patients with systolic heart failure, but it does not work rapidly. With a half-life of 36-48 hours in adults, it takes 7-10 days for the serum digoxin levels to reach steady-state concentration. Intravenous loading doses (which are sometimes used to achieve rate control in patients with atrial fibrillation) are not used in patients with CHF.
ACE inhibitors such as enalapril (option B) and certain β-blockers such as metoprolol (option C) have been proven to reduce mortality in patients with systolic heart failure, but neither is indicated for the acute treatment of ADHF and pulmonary edema. In fact, β-blockers are relatively contraindicated in patients with ADHF since they depress myocardial contractility. β-blockers should be started after the patient has been stabilized and must be titrated cautiously to the maximum tolerated dose.
Other treatments that might be appropriate for this patient include supplemental oxygen (because of his low oxygen saturation) and possibly noninvasive positive pressure ventilation (if there is evidence of respiratory distress or respiratory acidosis).
Question:
Answer: C. This patient most likely has a subarachnoid hemorrhage. The classic history is of a sudden, severe headache (often described as the worst headache of the patient’s life), which may be accompanied by altered mental status, nausea and vomiting, loss of consciousness, or meningismus. Most subarachnoid hemorrhages are caused by rupture of a saccular aneurysm (also known as a berry aneurysm).
Subarachnoid hemorrhage carries a mortality of nearly 50%, making prompt diagnosis and treatment essential. Non-contrast CT of the head is the best initial test for diagnosing an intracranial hemorrhage, because fresh blood will cause a bright white density on the scan. Small hemorrhages, however, may not be apparent on a CT, so if there is a strong clinical suspicion the most appropriate next test would be a lumbar puncture (option C). Patients with subarachnoid hemorrhage may have CSF that is grossly bloody, contains a large number of RBCs, or is pink or yellow in color (called “xanthochromia”) due to degradation of hemoglobin.
Alteplase (option A) is a fibrinolytic drug used to treat acute thrombotic stroke. Administering a thrombolytic drug to a patient with an acute subarachnoid hemorrhage could be catastrophic. This patient has no focal neurological deficits to suggest a stroke.
This patient does have signs and symptoms suggestive of meningitis (e.g., headache, lethargy, nausea, photophobia, meningismus), but most adults with meningitis will present with fever. The history of sudden, severe headache is more worrisome for subarachnoid hemorrhage than meningitis. It is also generally inappropriate to give empiric antibiotics (option B) for meningitis before performing a lumbar puncture, although antibiotics may be given immediately after the LP is completed, even before the results are available.
An MRI of the brain (option D) would be appropriate if you suspected an acute thrombotic stroke or a mass lesion, or MRI might be used instead of CT to diagnose an acute hemorrhage. But since this patient has already had one negative imaging study, a lumbar puncture would be the more appropriate choice.
Naproxen (option E) would be an appropriate treatment for many kinds of headache, such as tension headache. Sumatriptan (option D) would be an appropriate treatment for a migraine or cluster headache. But this case contains several so-called “red flags” (including sudden-onset of headache, age > 50, and meningismus) that should prompt further workup of the headache before initiating treatment.
Question:
Answer:
This patient presents with a triad of fever, neck stiffness, and altered mental status, which is highly suggestive of meningitis. The petechial rash developing into ecchymoses or purpura is suggestive of bacteremia due to Neisseria meningitidis, a Gram-negative diplococcus (Option F). N. meningitidis sepsis also increases the risk of purpura fulminans, a condition of widespread skin necrosis due to thrombosis within the cutaneous vasculature. Because of the highly contagious (and potentially devastating) nature of meningococcal meningitis, a meningococcal vaccine is recommended to all children at age 11-12 with a booster at age 16.
- Option A suggests Mycobacterium tuberculosis.
- Option B suggests Cryptococcus neoformans.
- Option C suggests Streptococcus pneumoniae.
- Option D suggests staphylococcal meningitis.
- Option E suggests Listeria monocytogenes.
- Option G suggests Escherichia coli.
- Option H could suggest either Borrelia burgdorferi or Treponema pallidum, which may cause both meningitis and cutaneous findings but which would not give this clinical picture.
- Option I would suggest viral meningitis due to a viral illness.
Question:
A 63-year-old woman comes to the physician with progressive muscle weakness, ataxia, dysarthria, and dysphagia. Amyotrophic lateral sclerosis is eventually diagnosed. Which of the following findings would most likely be seen in this patient?(A) Atrophy of the caudate on MRI
(B) Elevated CSF protein with normal CSF white blood cell count
(C) Improvement in muscle strength following administration of edrophonium
(D) Inappropriate laughter
(E) Prominent sensory deficits
Answer: D. Amyotrophic lateral sclerosis (ALS) is a progressive neurodegenerative disease marked by the presence of both upper and lower motor neuron signs and symptoms. 20% of patients will present initially with bulbar muscle dysfunction, resulting in dysarthria and/or dysphagia. As the disease progresses, involvement of the respiratory muscles is common. The sensory examination (option E) is typically normal in individuals with ALS. While some 20-30% of ALS patients may report paresthesias or other sensory symptoms, they usually deny loss of sensation.Mood-incongruent outbursts of laughter, crying or yawning may be seen as an early or late manifestation of ALS (option D). This “pseudobulbar affect” is thought to be due to lesions of the descending corticobulbar tracts.
Atrophy of the caudate nuclei in the basal ganglia (option A) is characteristic of patients with Huntington disease, which is characterized by choreiform movements and cognitive dysfunction.
Elevated CSF protein with normal CSF white blood cell count (option B) is called “albuminocytologic dissociation”, and is seen in patients with Guillain-Barré syndrome.
Improvement in muscle strength following administration of edrophonium (option C) is characteristic of myasthenia gravis.
Question:
A 52-year-old man comes to the physician because of muscle weakness. It began as weakness in the hands, which has progressively worsened over the past two months. He has noticed a sensation of “heaviness” in his legs and increasing worsening of his balance when walking. His speech has become slurred and difficult to understand. Physical examination shows muscle atrophy and fasciculations in the hands and the calves. The patient has difficulty rising from his chair and climbing onto the examination table. Handgrip strength is 1/5. Motor strength is 2/5 in the other major muscle groups of the upper and lower extremities. The sensory examination is normal. Biceps, patellar, and Achilles reflexes are all 3+. The Babinski reflex is positive bilaterally. Which of the following is the most likely course of this patient’s disease?
(A) Complete spontaneous recovery
(B) Complete recovery after treatment with riluzole
(C) Periodic remission of symptoms followed by relapses
(D) Progressive deterioration with death within 3-5 years
(E) Survival without reversal of symptoms after treatment with riluzol
Answer: D. The patient’s presentation of upper and lower motor neuron findings is consistent with amyotrophic lateral sclerosis, a progressive neurodegenerative disease with a median survival of 3-5 years after diagnosis. The 10-year survival rate is approximately 10-20%. The rate of progression varies from patient to patient, but the clinical course does not involve the exacerbations, remissions, and relapses that are often seen in diseases such as multiple sclerosis. Therapy for ALS is primarily focused on treatment of symptoms and palliative care. The drug riluzole modestly slows disease progression but does not reverse symptoms or provide a cure.
Question:A 26-year-old woman comes to the physician because of acne unresponsive to over-the-counter topical treatments. She also wishes to discuss weight loss, saying that she has tried dieting and exercising but has been unable to lose weight. Upon further questioning, she reveals that she routinely shaves hair off her upper lip and chin. Menses occur at irregular 45- to 90- day cycles. She is 160 cm (5 ft 3 in) tall and weighs 77 kg (170 lb); BMI is 30 kg/m2. An ultrasound of the pelvis shows enlarged ovaries with multiple peripheral follicles bilaterally.
Question:
Which of the following medications is considered first-line pharmacotherapy for this patient’s condition?
(A) Clomiphene
(B) Leuprolide
(C) Metformin
(D) Combination OCPs
(E) Paragard IUD
(F) Spironolactone
Answer: (D) Combination OCPs. This patient most likely has polycystic ovarian syndrome (PCOS). According to the Rotterdam criteria, the diagnosis of PCOS is made if a patient has at least 2 of the following:
- Oligo- or anovulation
- Evidence of hyperandrogenism
- Polycystic ovaries on ultrasound
Combination OCPs are the first-line pharmacological treatment for PCOS. Benefits include regulation of menses, protection against endometrial hyperplasia, improvement in hyperandrogenic symptoms, and contraception. Spironolactone (F) has anti-androgenic effects so is sometimes added to the treatment regimen if hirsutism continues to be a problem. Metformin (C) is often used as a second-line therapy in PCOS, which is associated with glucose intolerance and insulin resistance. Clomiphene (A) and pulsatile leuprolide (B) are used to induce ovulation in patients with PCOS who desire pregnancy. The Paragard IUD (G), which is the copper-containing IUD, would provide no additional benefit to this patient beyond contraception.
Question:
A 32-year-old wildlife biologist studying in Uganda is brought to the hospital by his research assistant because of a 2-hour history of obtundation. His research assistant says that the patient had developed fever, chills, and malaise one week after performing a necropsy on a dead chimpanzee. His temperature is 40°C (104°F), pulse is 130/min, and blood pressure is 90/60 mm Hg. Physical examination shows bilateral conjunctival hemorrhages, multiple bruises, and a generalized non-pruritic maculopapular rash. Twenty minutes after a blood sample is obtained, blood continues to ooze from the venipuncture site despite continuous pressure held by the nurse. A non-segmented, negative-sense single-stranded RNA virus is isolated from the patient’s blood. An electron micrograph of the virus is shown. This virus most likely belongs to which of the following viral families?
(A) Arenavirus
(B) Bunyavirus
(C) Coronavirus
(D) Flavivirus
(E) Filovirus
(F) Paramyxovirus
(G) Rhabdovirus
(H) Togavirus
Answer: (E) Filovirus. This patient most likely has hemorrhagic fever due to infection with the Ebola virus, a member of the Filovirus family, so named because of the thread-like shape of the virion. Found primarily in sub-Saharan Africa, the Ebola virus causes occasional outbreaks in apes and humans; the mortality rate approaches 90% in some of these outbreaks, depending on the strain. The current outbreak affecting Guinea, Liberia, Sierra Leone, and now Nigeria is the worst Ebola outbreak in history, with more than 1,000 deaths at this point. The virus is spread by exposure to infected bodily fluids. Upon entering the body, the virus first infects macrophages and dendritic cells, which release cytokines and other proinflammatory mediators, leading to a widespread inflammatory response and eventually shock. Coagulopathy results in the bleeding diathesis for which the Ebola virus is famous.
Question:
A 68-year-old African American woman comes to the physician complaining of fatigue and shortness of breath during her weekly trips to the grocery store. She has a history of hypertension. Physical examination shows mild pedal edema. Her lungs are clear. An echocardiogram shows an estimated left ventricular ejection fraction of 35% without valvular dysfunction. A drug with which of the following mechanisms of action would be most likely to reduce mortality in this patient?
(A) Agonist of β1 adrenergic receptors
(B) Antagonist of aldosterone receptors
(C) Inhibits conversion of angiotensinogen to angiotensin I
(D) Inhibits Na+/K+-ATPase
(E) Inhibits Na+-K+-Cl- co-transporter in the loop of Henle
Answer: This patient has compensated congestive heart failure, with symptoms only during exertion. Three groups of drugs are proven to reduce mortality in patients with systolic heart failure:
- ACE inhibitors (and certain angiotensin receptor-blockers)
- β-blockers (bisoprolol, carvedilol, and extended-release metoprolol)
- Aldosterone antagonists (spironolactone and eplerenone)
Option B is therefore the correct choice.
- β1 agonists (option A) such as albuterol are not used to treat CHF.
- Option C describes the mechanism of action of the direct renin inhibitor aliskiren, which inhibits conversion of angiotensinogen to angiotensin I. Aliskiren, however, is not used to treat CHF. ACE inhibitors, which are useful in treating CHF, work by inhibiting the conversion of angiotensin I to angiotensin II.
- Digoxin inhibits Na+/K+-ATPase (option D) and is proven to improve symptoms and reduce hospitalizations in CHF patients, but not to reduce mortality.
- Likewise, loop diuretics (option E) such as furosemide and bumetanide are useful in reducing symptoms of volume overload in CHF patients but have not been proven to reduce mortality.
Question:
A 21-year-old man comes to the emergency department because of a 1-hour history of severe left scrotal pain, nausea, and vomiting. He says that the pain started suddenly, awakening him from sleep. He is sexually active and uses condoms for contraception inconsistently. His temperature is 37°C (98.6°F), pulse is 110/min, and blood pressure is 120/80 mm Hg. Physical examination shows a high-riding left testis. Stroking of the right upper thigh results in elevation of the right testis; there is no response when the left upper thigh is stimulated. When the left testis is rotated laterally, there is complete resolution of the pain and the testis returns to normal position and orientation in the scrotum.Which of the following is the most appropriate next step in management?
(A) Administration of antibiotics (ceftriaxone + doxycycline)
(B) Doppler ultrasonography of the scrotum
(C) MRI of the scrotum
(D) Reassurance and outpatient follow-up
(E) Surgical consultation
Reference
Answer: (E) Surgical consultation. This patient most likely had testicular torsion, suggested by the acute onset of scrotal pain, the high-riding testis, and the absent cremasteric reflex on the affected side. While manual detorsion was apparently successful, with resolution of the signs and symptoms, this patient still needs surgical consultation in order to secure the testicle to the scrotal wall (i.e., orchiopexy) to prevent future episodes of torsion. While Doppler ultrasonography is the test of choice in diagnosing testicular torsion, its usefulness would likely be limited in this patient after blood flow to the testis was restored.
Question:
Answer: C - CT scan of the chest. This patient’s presentation is consistent with hypercortisolism (Cushing syndrome), which causes purple abdominal striae, moon facies, hirsutism, truncal obesity, easy bruising, proximal muscle wasting, and other findings. The most common cause of Cushing syndrome is exogenous glucocorticoid use. The second most common cause is thought to be ectopic production of adrenocorticotropic hormone (ACTH) from a non-pituitary site, such as a small cell lung cancer. Less common causes of hypercortisolism include Cushing disease (ACTH production from a functioning pituitary adenoma) and primary hypercortisolism (from an adrenal adenoma).
This patient has high serum levels of cortisol and ACTH, which rule out iatrogenic Cushing syndrome from exogenous glucocorticoids and which suggest an ACTH-dependent Cushing syndrome. High serum ACTH also rules out primary hypercortisolism.
The low-dose (1 mg) and high-dose (8 mg) dexamethasone suppression tests, in which dexamethasone is given at night and the serum cortisol level is checked the following morning, also yield consistently high serum cortisol levels in this patient, indicating ectopic ACTH secretion by non-pituitary tissue that does not respond to normal feedback inhibition by circulating glucocorticoids.
In a smoker with suspected ectopic ACTH production, imaging of the chest would be an appropriate diagnostic test; therefore (C) CT scan of the chest is the correct response. An MRI of the brain to look for a pituitary mass would not be helpful in this patient, as pituitary production of ACTH will usually be suppressed with a high-dose dexamethasone suppression test. A cosyntropin stimulation test would be appropriate for a patient with suspect primary adrenal insufficiency (Addison disease). Saline infusion testing and plasma aldosterone and renin levels would be used to diagnose the cause of hyperaldosteronism.
Question:
Answer:
The three shunts present in fetal circulation are:
- The ductus venosus, which shunts blood from the umbilical cord into the inferior vena cava, bypassing the liver.
- The foramen ovale, which shunts blood from the right atrium to the left atrium, bypassing the lungs.
- The ductus arteriosus also bypasses the lungs, shunting blood from the pulmonary artery to the aorta
A patent ductus arteriosus (PDA) can be closed with indomethacin. Prostaglandins E1 and E2 can be used to keep a PDA open, as might be necessary with certain congenital anomalies (e.g., transposition of the great vessels) until surgical intervention can be performed.
Question:
A 46-year-old man is brought to the emergency department by his friend because of confusion and difficulty walking. He is alert but not oriented to person, place, or time. His friend says that the patient has been living on the streets since losing his home 5 years ago. He appears disheveled. Physical examination shows jaundice, a distended abdomen, and 2+ edema of the lower extremities. Neurologic examination shows horizontal nystagmus, impaired abduction of both eyes, and difficulty performing tandem gait. There is no asterixis. Which of the following is the most likely cause of these findings?
(A) Bacterial meningitis
(B) Chronic subdural hematoma
(C) Concussive head injury
(D) Hepatic encephalopathy
(E) HSV encephalopathy
(F) PCP intoxication
(G) Temporal lobe infarction
(H) Wernicke encephalopathy
Answer: (H) Wernicke encephalopathy (WE). A result of thiamine deficiency, WE is a neurologic condition seen most commonly in patients with chronic alcoholism and/or malnutrition. The classic triad includes encephalopathy, ophthalmoplegia, and ataxia; however, only about 1/3 of cases feature all 3 components of the triad. WE can progress to coma and even death, so physicians should maintain a high index of suspicion in patients with evidence of long-term alcohol abuse or malnutrition and any of the following symptoms: ataxia, ophthalmoplegia, acute confusion, memory disturbance, hypothermia with hypotension, or delirium tremens. The treatment for WE is intravenous thiamine. Importantly, thiamine must be administered before glucose in any patient susceptible to WE, as giving glucose before thiamine can actually precipitate WE.
Question:
Answer: D. The lesion shown is consistent with oral hairy leukoplakia, a lesion of the tongue and oral mucosa caused by Epstein-Barr virus (EBV), especially in patients with concurrent HIV infection. The lesions are typically found on the lateral surface of the tongue and cannot be scraped off (unlike oral thrush, caused by Candida albicans). “Oral hairy leukoplakia” is distinct from “oral leukoplakia,” a precancerous inflammatory lesion usually found on the dorsum of the tongue associated with smokeless tobacco use and HPV infection.
Question:
Answer: E. This patient is suffering from rabies encephalitis, which is usually contracted through the bite of an infected animal. After a one-week prodromal period of fever, malaise, headache, sore throat, and nausea/vomiting, the patient develops delirium and agitation, muscle spasms, and autonomic instability (tachycardia, diaphoresis, lacrimation, hypersalivation). Many patients develop dysphagia and a profound fear of drinking water (i.e., hydrophobia), since attempts to drink can provoke involuntary pharyngeal spasms.
Eosinophilic neuronal cytoplasmic inclusions called Negri bodies in the brain tissue are pathognomonic for rabies but are not always present.
Question:
Answer: (B) IV amphotericin B + oral flucytosine. This patient most likely has meningitis due to infection with Cryptococcus neoformans. Cryptococcal meningitis is most commonly seen in patients with impaired immune systems; this patient’s CD4+ T-lymphocyte count is consistent with severe immunosuppression due to AIDS. The photomicrograph shows the round encapsulated C. neoformans yeast forms that can be visualized with India ink staining. The recommended initial treatment for cryptococcal meningitis is dual therapy with IV amphotericin B + oral flucytosine. After a period of improvement, the patient can then be switched to oral fluconazole for the remainder of the necessary treatment.
Question:
Answer: Image “A” demonstrates a sliding (type I) hiatal hernia and image “B” demonstrates a paraesophageal (type II) hiatal hernia. The more common of the two types, sliding hiatal hernias (“A”), occur when the gastroesophageal junction (GE junction) herniates into the thorax through the esophageal hiatus. In contrast, paraesophageal hiatal hernias (“B”) involve herniation of part of the stomach through the esophageal hiatus with no displacement of the GE junction.
Acid reflux is more commonly associated with sliding hiatal hernias (type I), due to superior displacement of the GE junction.
Question:
Answer: C – Decreases ATP production and inhibits polymerization of a component of the microtubules. This patient most likely has neurocysticercosis. Neurocysticercosis is caused by ingestion of the pork tapeworm Taenia solium. Clinical manifestations include: seizures, headache, nausea, and vomiting. The drug of choice for the treatment of active neurocysticercosis is albendazole. Albendazole decreases ATP production in the parasite and inhibits polymerization of a component of the microtubules.
Question:
Answer: I - Plummer-Vinson syndrome. This patient most likely has Plummer-Vinson syndrome. This is classically characterized by the triad of dysphagia (primarily to solids), iron deficiency anemia, and cervical esophageal webs. An upper GI series is usually performed to confirm the diagnosis. Additionally, signs of iron deficiency anemia may be present (e.g., weakness, fatigue, koilonychia, glossitis, etc.). Etiology is unknown, and treatment consists primarily of iron supplementation.
Question:
For the image shown above, answer the following questions:
1) What is the most likely diagnosis?
2) In what region of the United States is this condition most commonly found?
3) What pathogen causes this condition?
4) What is the preferred treatment for this condition?
5) Which cranial nerve is most commonly affected as this condition progresses?
Answer:
1) The image shows the classic “bull’s-eye” rash (erythema migrans) which is hallmark feature of Lyme disease.
2) Lyme disease is endemic to the northeastern region of the United States. 13 states account for over 95% of all cases of Lyme disease. These states include: Connecticut, Massachusetts, New Jersey, New York, Pennsylvania, Maryland, Delaware, Virginia, Maine, New Hampshire, Vermont, Minnesota, and Wisconsin.
3) Lyme disease is caused by Borrelia burgdorferi which is transmitted by the Ixodes tick.
4) The drug of choice for the treatment of Lyme disease is doxycycline.
5) CN-7 (facial nerve) can be affected in Lyme disease and results in facial nerve palsy. A person presenting with acute-onset facial nerve palsy (from an endemic region for Lyme disease), should strongly be suspected of having contracted Lyme disease.
Question:
For the image shown above, answer the following questions:
1) What is the most likely diagnosis?
2) What autosomal dominant (AD) condition is most commonly associated with this finding? What other clinical findings would be seen in a patient with this condition?
3) What autosomal recessive (AR) condition is most commonly associated with this finding? What other clinical findings would be seen in a patient with this condition?
Answer:
1) The image shows lens subluxation (or ectopia lentis). Subluxation refers to a lens that is partially dislocated but still attached to the ciliary body.
2) Lens subluxation is a common clinical finding in patients with Marfan syndrome. Marfan syndrome is most commonly inherited in an autosomal dominant fashion. In about two-thirds of Marfan cases, subluxation of the lens is SUPERIOR. Additional clinical findings that may be seen in Marfan syndrome include: 1) aortic disease (aortic regurgitation and aortic dissection), 2) mitral valve prolapse (MVP), 3) tall stature, 4) arm span greater than height, 4) arachnodactyly (long spider-like fingers), 5) pectus carinatum/pectus excavatum, and 6) scoliosis/kyphosis.
3) Lens subluxation is also seen in patients with Homocystinuria. Homocystinuria is most commonly inherited in an autosomal recessive fashion. In about two-thirds of Homocystinuria cases, subluxation of the lens is INFERIOR. Additional clinical findings that may be seen in Homocystinuria include: 1) intellectual disability, 2) osteoporosis, 3) tall stature, 4) kyphosis, 5) thrombosis, and 6) atherosclerosis.
Question:
Answer: A – Age-related macular degeneration. Age-related macular degeneration (AMD) is an ophthalmologic disease that involves degeneration of the central portion of the retina (the macula) and gradually results in loss of central vision. Patients will commonly complain of difficulty in reading, driving, and other activities requiring the use of central vision. Patients will become increasingly dependent on magnifying lenses to assist in reading or driving. Several risk factors for AMD have been documented including: age, family history, diet, and cigarette smoking.
Question:
A 58-year-old man is brought to the emergency department after an attempted suicide. He was recently diagnosed with major depressive disorder. He is coherent and complains of excruciating retrosternal chest and upper abdominal pain, odynophagia, and tachypnea. His temperature is 38.3°C (100.9°F). Physical examination shows subcutaneous emphysema and crepitation over the left anterior chest wall. A chest x-ray shows mediastinal and free peritoneal air. Which of the following is the most likely diagnosis? [Select the SINGLE BEST answer from the answer choices provided]:
A. Boerhaave syndrome
B. Esophagitis
C. Hiatal hernia
D. Mallory-Weiss tear
E. Perforated duodenal ulcer
F. Perforated gastric ulcer
G. Ruptured esophageal varices
H. Spontaneous pneumothorax
I. Tension pneumothorax
Answer: A. Boerhaave syndrome is a spontaneous perforation of the esophagus that most commonly results from a sudden increase in intraesophageal pressure combined with negative intrathoracic pressure caused by straining or vomiting. Other causes of spontaneous perforation include caustic ingestion (e.g., ingestion of lye; most likely in this case), pill esophagitis, Barrett's ulcer, infectious ulcers (e.g., AIDS patients), and iatrogenic (e.g., upper endoscopy – dilation of esophageal strictures). The condition is associated with high morbidity and mortality and requires immediate surgical intervention.
Question:
For the condition shown, answer the following questions:
1) What is a common physical exam finding which supports diagnosing a patient with this condition?
2) Name an extracutaneous comorbidity associated with this condition?
3) What is the most common form of the condition shown?
Answer:
1) Ausptiz sign, which refers to pinpoint (or punctate) bleeding that occurs when psoriatic scales are scraped off, is a finding supporting a diagnosis of psoriasis.
2) Examples of extracutaneous disorders that have been linked to psoriasis include: psoriatic arthritis, obesity, metabolic syndrome, autoimmune disease, nonalcoholic fatty liver disease, COPD, and obstructive sleep apnea.
3) The most common form of psoriasis is psoriasis vulgaris.
Question:
For the image shown, answer the following questions:
1) A patient presents with chorea, muscle rigidity, and dementia. Which of the labeled region(s) is/are most likely affected?
2) What is the mode of inheritance of this disease?
3) A mutation in which chromosome is implicated in this disease?
4) Which neurotransmitter(s) is/are decreased in this disease?
Answer:
1) Huntington disease (HD) results in atrophy (or loss) of striatal neurons in the caudate (B) and putamen (C).
2) HD is an autosomal dominant disease.
3) The HD gene is located on chromosome 4 and codes for the huntingtin protein.
4) Decreased levels of GABA and ACh are associated with HD.
Question:
For the image shown, answer the following questions:
1) Based on the condition shown in the image, what is the most likely diagnosis?
2) What characteristic finding is seen on serum protein electrophoresis (SPEP)?
3) What characteristic finding is seen on a peripheral blood smear?
4) Where is the most common site of bone pain seen in this condition?
5) BONUS: What is the mechanism for the bone lesions seen in this condition?
Answer:
1) The x-ray shows characteristic lytic (“punched out”) bone lesions. The most likely diagnosis is multiple myeloma (MM).
2) An M-spike on SPEP is typically seen and is a classical finding in MM.
3) Roleaux formation (RBCs stacked like coins) is typically seen on a peripheral blood smear in MM.
4) The vertebra is the most common site of bone pain seen in MM. Other sites include the ribs, skull, and pelvis.
5) BONUS: The multiple lytic bone lesions seen in MM are due to osteoclastic activating factor (OAF); this results in hypercalcemia.
Question:
For the image shown, answer the following questions:
1) Identify the labeled structures (A-G).
2) What is the name of the sphincter that controls the structure labeled (L)?
3) What life-threatening complication can develop as a result of a gallstone becoming lodged within the structure labeled (L)?
4) BONUS: Which hormone stimulates the release of bile from the gallbladder?
Answer:
1) (A) Right hepatic duct, (B) left hepatic duct, (C) cystic duct, (D) common hepatic duct, (E) common bile duct, (F) accessory pancreatic duct, and (G) pancreatic duct.
2) The sphincter responsible for controlling the major duodenal papilla (ampulla of Vater) (L) is the sphincter of Oddi.
3) Gallstone-induced pancreatitis can result from a gallstone that becomes lodged within the major duodenal papilla (ampulla of Vater) (L).
4) Cholecystokinin (CCK), which is produced by I cells of the duodenum and jejunum, stimulates the gallbladder to contract and release bile.
Question:
A 63-year-old man develops paresthesias and ataxia. He has a history of hypertension, type 2 diabetes, gastroesophageal reflux disease, hypercholesterolemia, and glaucoma. He drinks a glass of wine every night with dinner. Laboratory studies show elevated levels of plasma homocysteine. Physical examination shows a positive Romberg test. Which of the following is the most likely etiology for this patient’s symptoms? [Select the SINGLE BEST answer from the answer choices provided]:
- Alcohol
- Enalapril
- Metformin
- Omeprazole
- Simvastatin
- Timolol
Answer: D – Omeprazole. Prolonged use of proton pump inhibitors (e.g. omeprazole) has been strongly associated with vitamin B12 deficiency. B12 deficiency results as a consequence of impaired release of cobalamin from food in the absence of gastric acid secretion.
Question:
For the image shown, answer the following questions:
1) What is the strongest risk factor for the development of this finding?
2) What complication is associated with this finding?
3) What is the imaging test of choice used to confirm diagnosis of this condition?
4) BONUS: Which endocrine disorder is paradoxically associated with a decreased risk for developing this condition?
Answer:
1) Several risk factors have been associated with abdominal aortic aneurysm (AAA) including: 1) smoking, 2) male gender, 3) advancing age, 4) Caucasian race, 5) atherosclerosis, and 6) hypertension. Among these, smoking is the strongest risk factor for the development of an AAA. In fact, smokers are 8 times more likely to develop an AAA compared to non-smokers.
2) The most common (and feared) complication of an AAA is rupture.
3) For nonruptured AAA, the imaging test of choice is abdominal ultrasonography.
4) Diabetes, along with female gender and non-Caucasian race, are associated with a decreased risk of developing an AAA.
Question:
A 23-year-old woman has a 2-day history of dysuria, frequency, urgency, and suprapubic pain. She is sexually active and uses an oral contraceptive. Her temperature is 36.6°C (98°F). Urinalysis shows a positive nitrite test. What is the mechanism of action of the drug most likely used to treat this condition? [Select the SINGLE BEST answer from the answer choices provided]
- Alters the permeability of the cell wall by blocking fungal cytochrome P450
- Blocks peptide transfer (transpeptidation) at the 50S ribosomal subunit
- Binds ergosterol and forms membrane pores that allow leakage of electrolytes
- Binds to 30S and prevents attachment of aminoacyl-tRNA
- Forms free radical toxic metabolites in the bacterial cell that damage DNA
- Inhibits bacterial cell wall synthesis by blocking glycopeptide polymerization through binding to D-alanyl-D-alanine portion of cell wall precursor
- Inhibits nucleotide binding to reverse transcriptase and terminates the DNA chain
- Inhibits viral DNA polymerase by acting as an analog to deoxyguanosine triphosphate (dGTP)
- Interferes with bacterial folic acid synthesis and growth via inhibition of dihydrofolic acid formation
- Reversibly binds to 50S ribosomal subunits preventing amino acids from being transferred
Anwer: 9 – interferes with bacterial folic acid synthesis and growth via inhibition of dihydrofolic acid formation. Based on the clinical presentation, this patient most likely has an uncomplicated urinary tract infection (UTI). The microbial spectrum of uncomplicated cystitis and pyelonephritis in women consists mainly of Escherichia coli (75%-95%), with occasional other species. Other gram-negative and gram-positive species are rarely isolated in uncomplicated UTIs. From the drugs listed, TMP-SMX (Bactrim) is the single best choice for the treatment of an uncomplicated UTI. Sulfamethoxazole interferes with bacterial folic acid synthesis and growth via inhibition of dihydrofolic acid formation from para-aminobenzoic acid; trimethoprim inhibits dihydrofolic acid reduction to tetrahydrofolate resulting in sequential inhibition of enzymes of the folic acid pathway.
Question:
For the image shown, answer the following questions:
1) Identify the pathology associated with each of the labeled structures (A, B, C, and D).
2) What is the most likely diagnosis?
3) Which of the labeled structures (A, B, C, or D) is the most important determinant for prognosis?
4) BONUS: What murmur would most likely be heard on cardiac auscultation?
Answer:
1) (A) Overriding aorta, (B) VSD, (C) RVH, and (D) Pulmonary infundibular stenosis
2) Tetralogy of Fallot (TOF)
3) (D) – The most important determinant for prognosis is the severity and degree of pulmonic stenosis in a patient with TOF.
4) The murmur is typically crescendo-decrescendo with a harsh systolic ejection quality; it is appreciated best along the left mid to upper sternal border with radiation posteriorly.
Question:
A 40-year-old woman comes to the emergency department with a 6-hour history of colicky abdominal pain, nausea, and vomiting. Her temperature is 36.6°C (98°F), pulse is 110/min, and blood pressure is 95/75 mmHg. Physical examination shows dry mucous membranes, abdominal distention, and three well-healed 10-12 mm pelvic scars. On abdominal auscultation hypoactive and muffled bowel sounds are heard. Which of the following is the most likely etiology of the abdominal pain? [Select the SINGLE BEST answer from the answer choices provided]:
- Acute cholecystitis
- Acute mesenteric ischemia
- Acute pancreatitis
- Appendicitis
- Celiac disease
- Colon cancer
- Crohn disease
- Direct inguinal hernia
- Ectopic pregnancy
- Femoral hernia
- Hiatal hernia
- Incarcerated hernia
- Incisional hernia
- Indirect inguinal hernia
- Intestinal volvulus
- Intussusception
- Large bowel obstruction
- Meckel diverticulum
- Ovarian torsion
- Peritonitis
- Periumbilical hernia
- Small bowel obstruction
- Ulcerative colitis
- Ventral hernia
Answer: V – small bowel obstruction (SBO). SBO occurs when the normal flow of intraluminal contents is interrupted. The obstruction can be functional (due to abnormal intestinal physiology) or due to a mechanical obstruction, which can be acute or chronic. The most common causes of mechanical SBO are postoperative adhesions and hernias. The most important risk factor for mechanical SBO is prior abdominal surgery causing postoperative adhesions. Patients with a history of prior abdominal or pelvic surgery, and particularly colorectal surgery, appendectomy, gynecologic surgery, prior adhesiolysis, and resection of malignancy are prone to adhesive SBO.
Question:
A patient has the karyotype shown below:
Would you expect the serum concentrations of the following hormones to be above normal, below normal, or unchanged from normal?
- Testosterone
- Luteinizing hormone (LH)
- Follicle-stimulating hormone (FSH)
Answer: Testosterone (BELOW NORMAL), LH (ABOVE NORMAL), and FSH (ABOVE NORMAL). Klinefelter syndrome (47, XXY) is the most common congenital abnormality causing primary hypogonadism. The distinction between primary and secondary hypogonadism is made by measurement of the serum concentrations of luteinizing hormone (LH) and follicle-stimulating hormone (FSH): The patient has primary hypogonadism if the serum testosterone concentration and the sperm count are below normal and the serum LH and FSH concentrations are above normal. The patient has secondary hypogonadism if the serum testosterone concentration and the sperm count are below normal and the serum LH and FSH concentrations are normal (or below normal).
Question:
An 8-year-old boy has an episode of bloody diarrhea after attending a family picnic. 2-weeks later, he develops an unsteady gait and pain and numbness in the toes and fingertips. Physical examination shows symmetric weakness with absent deep tendon reflexes. Electrophysiologic studies show partial motor conduction block and slowed nerve conduction velocities. Which of the following treatments should most likely be initiated? [Select the SINGLE BEST answer from the answer choices provided]
A. Azithromycin
B. Ceftriaxone
C. Ciprofloxacin
D. Gentamicin
E. IV immune globulin
F. IV vancomycin
G. Metronidazole
H. Neostigmine
I. Oral vancomycin
J. Penicillin
K. Systemic corticosteroids
Answer: E – IV immune globulin. Campylobacter enteritis is typically caused by Campylobacter jejuni or Campylobacter coli. The organism inhabits the intestinal tracts of a wide range of animal hosts, notably poultry; contamination from these sources can lead to foodborne disease. C. jejuni infection has been established as a trigger of GBS, an acute immune-mediated polyneuropathy. It has been estimated that 30-40% of GBS illness is attributable to Campylobacter infection, which typically occurs between 1 and 2 weeks before the onset of neurologic symptoms. First line agents for treatment of Campylobacter gastroenteritis include fluoroquinolones or azithromycin. Guillain-Barre syndrome (acute inflammatory demyelinating polyradiculoneuropathy) is the most common cause of acute flaccid paralysis in healthy infants and children. Approximately two-thirds of patients have a history of a previous respiratory tract or gastrointestinal infection. A variety of infectious agents have been associated with GBS, although Campylobacter jejuni is the most frequent. The classic presentation is fine paresthesias in the toes and fingertips followed by lower extremity symmetric weakness that may ascend over hours to days to involve the arms. The predominant symptoms of GBS at presentation in children are pain and gait difficulty. The main treatment modalities for GBS include plasmapheresis and administration of intravenous immune globulin.
Question:
In a patient with active peptic ulcer disease, what is the 1st line of treatment for the condition shown here?
Answer: Colchicine. Acute gout is an intensely painful and disabling inflammatory arthritis usually involving a single joint but occasionally involving two or more joints. The goal of therapy in an acute gout attack to address the pain and disability associated with it. Symptoms improve more quickly with administration of any of a broad array of anti-inflammatory drugs. However, if the use of an NSAID is contraindicated (e.g. active peptic ulcer disease, chronic kidney disease etc.), the next best treatment option is an oral low-dose colchicine regimen.
Question:
A 33-year-old woman comes to the physician. She states that she feels no pain in her hands and frequently burns them while cooking. She drinks 2-3 glasses of wine a day. Physical examination shows bilateral decreased pain and temperature sensation in the upper extremities, absent deep bilateral tendon reflexes in the upper extremities, and marked atrophy of the intrinsic muscles of the hands. An MRI is scheduled. Which of the following is the most likely diagnosis?
- Amyotrophic lateral sclerosis (ALS)
- Friedreich ataxia
- Guillain-Barre syndrome
- Multiple sclerosis
- Syringomyelia
- Tabes dorsalis (tertiary syphilis)
- Thiamine deficiency
- Vitamin B
Answer: E – Syringomyelia. Syringomyelia is a degenerative disease that produces a fluid-filled cavity (called a syrinx) in the cervical spinal cord, causing cervical cord enlargement. Most lesions are between C2 and T9. As the syrinx expands, it destroys the crossed lateral spinothalamic tracts (loss of pain and temperature sensation) and anterior horn cells (atrophy of intrinsic muscles of the hand).
Question:
Based on the peripheral blood smear shown, answer the following questions (with a SINGLE answer):
1. What is the most likely diagnosis?
2. What is the mode of inheritance? (e.g. autosomal dominant, autosomal recessive, etc.)
3. What is the most common type of anemia associated with this disease? (e.g. microcytic, macrocytic, etc.)
4. Which biochemical pathway is this deficient enzyme the rate-limiting enzyme for?
5. What type of cells are indicated by the red arrows in the image?
Answer:
1. Glucose-6-phosphate dehydrogenase (G6PD) deficiency
2. X-linked recessive (XLR)
3. Normocytic anemia
4. G6PD is the rate-limiting enzyme in the pentose phosphate pathway (PPP)
5. Heinz bodies are aggregates of denatured hemoglobin and are not normally present in RBCs. They are most commonly found in G6PD deficient people.
Question:
A 33-year-old morbidly obese woman comes to the physician to discuss weight loss. She is 167 cm (5 ft 6 in) tall and weighs 142 kg (315 lb); BMI is 50.8 kg/m2. Her blood pressure is 150/90 mmHg, respiratory rate is 24/minute, and pulse is 90 beats/minute. Which of the following is the most appropriate initial step in the management of this patient?
A. Admission to a eating disorders facility
B. Avoidance of alcohol
C. Avoidance of fatty foods
D. Behavior modification therapy
E. Cardiac stress test
F. Dietary and caloric intake modification
G. Dietary supplements
H. Diet and exercise counseling
I. ECG
J. Laparoscopic adjustable gastric band placement (Lap Band)
K. Liposuction
L. Gastrectomy
M. Gastric bypass surgery
N. Group counseling
O. Nutritional counseling
P. Pharmacologic intervention
Answer: H – Diet and exercise counseling. More than two-thirds of adults in the United States are either trying to lose weight or to maintain their weight. However, only 20% are both eating fewer calories and engaging in at least 150 minutes of physical activity during leisure time each week. Thus, clinicians can play an important role in educating people regarding the need for and the optimal strategies for losing weight. For initial weight loss, treatment should be aimed at decreasing food intake and, when possible, increasing energy expenditure. This can be accomplished by educating and counseling the patient on a proper diet (e.g. caloric intake, low fat foods, etc.) and introducing exercise into their daily routine. If this is unsuccessful, other approaches may need to be considered.
Question:
- What is the most common type of cancer associated with the structure labeled “A”?
- Functional loss or impairment of which of these structures most commonly results in gastroesophageal reflux disease? Identify this structure?
- Identify the lesser curvature.
- Which of these structures is utilized when performing a Nissen fundoplication (anti-reflux surgery)? Identify this structure?
- Parietal cells are responsible for the secretion of HCl and intrinsic factor (IF). Identify where parietal cells are located? Identify the structure.
- A 4-week-old baby develops postprandial, non-bilious, projectile vomiting. Which of these structures is associated with this pathology? What is the diagnosis?
- In a patient with chronic achalasia, a Heller myotomy is performed. Which of these structures is surgically excised during this procedure? Identify this structure.
Answer:
- (A) represents the distal esophagus and the most common cancer that occurs here is adenocarcinoma. Remember, Barrett’s esophagus is a pre-cancerous lesion and DOES NOT indicate active cancer; however, once Barrett’s esophagus has been diagnosed, the risk of developing adenocarcinoma greatly increases.
- (B) represents the lower esophageal sphincter (LES). GERD results from the impairment or dysfunction of the LES.
- (D) represents the lesser curvature of the stomach.
- (C) represents the fundus of the stomach. In performing a Nissen fundoplication, the fundus is circumferentially ‘wrapped’ around the LES to prevent gastric contents from refluxing into the distal esophagus; thereby, decreasing the effects of chronic acid-reflux disease.
- (E) represents the body of the stomach and greater than 80% of parietal cells are located here. The remaining 20% are found in the antrum (F) of the stomach.
- (G) represents the pyloric sphincter of the stomach. Hypertrophic pyloric stenosis is associated with non-bilious, projectile vomiting, in a newborn (minimum 2-weeks-old).
- The muscle fibers of the LES (B) are surgically excised while performing a Heller myotomy. A Heller myotomy is commonly performed for lower esophageal disorders (such as achalasia).
Question:
A 16 year old boy comes to the emergency department with a 3 hour history of severe knee pain. He is sexually active and admits to IV drug abuse. Physical examination shows a temperature of 39.9° C (102°F) and an erythematous, warm, tender and swollen right knee. Genetic testing shows a single nucleotide substation of valine for glutamic acid at the sixth position of the β-globin chain. Which of the following is the most common cause for this patient’s symptoms?
- Chlamydiae
- Enterobacter cloacae
- Escherichia coli
- Haemophilus influenzae
- Klebsiella pneumoniae
- Mycobacterium tuberculosis
- Neisseria gonorrhoeae
- Pasteurella multocida
- Plasmodium falciparum
- Proteus mirabilis
- Pseudomonas aeruginosa
- Salmonella paratyphi
- Salmonella typhi
- Shigella
- Staphylococcus aureus
- Staphylococcus saprophyticus
- Serratia marcescens
- Streptococcus pneumoniae
- Streptococcus pyogenes
Answer: L – Salmonella paratyphi. Although sickle cell disease (SCD) is characterized by a marked heterogeneity in clinical and hematologic severity, bone and joint problems are the most common manifestations. Pain, caused by vaso-occlusive crises, may occur as often as every week or as rarely as once a year. Because of the early decrease in splenic function, bacterial infections of bone and joints are common in children with SCD. Areas of infarcted bone or bone marrow are typical sites for infections. Although blood cultures during septic events most commonly yield Streptococcus pneumoniae or Haemophilus influenzae, osteomyelitis is usually due to Salmonella. Staphylococcus aureus, the most common cause of osteomyelitis in normal hosts, probably accounts for only one-fourth of all cases in SCD.
Question: What is the most likely diagnosis?
Answer: Dextrocardia (secondary to Kartagener’s/immotile-cilia syndrome). Although the CXR may appear to be a flipped or reversed film, recall that situs inversus is a manifestation of Kartagener’s; therefore, the liver is located on the patient’s left side resulting in a slightly higher left diaphragm as indicated on the CXR.
Question:
A 47-year-old man comes to the emergency department with a 3-hour history of abdominal pain, nausea, vomiting, dyspnea, decreased appetite, and fatigue. He has a past medical history of asthma, diabetes, hyperlipidemia, and hypertension. Physical examination shows severe tenderness of the upper abdomen and epigastric pain radiating to the back, ecchymotic discoloration in the periumbilical region, temperature of 40°C (104°F), and respirations of 35/min. The pain is partially relieved by sitting up or bending forward. Laboratory studies show slight leukocytosis, normal serum amylase and lipase, and elevated trypsinogen activation peptide. Which of the following is the most likely diagnosis?
- Acute cholecystitis
- Acute hepatitis
- Acute interstitial edematous pancreatitis
- Acute pancreatitis
- Appendicitis
- Cholangitis
- Choledocholithiasis
- Chronic pancreatitis
- Diverticulitis
- Intestinal obstruction
- Intussusception
- Mesenteric ischemia
- Necrotizing pancreatitis
- Peptic ulcer disease
Answer: C - acute interstitial edematous pancreatitis. The diagnosis of acute pancreatitis requires the presence of 2 of the following 3 criteria: 1) acute onset of persistent, severe, epigastric pain often radiating to the back, 2) elevation in serum lipase or amylase to 3 times or greater than the upper limit of normal, and 3) characteristic findings of acute pancreatitis on imaging (contrast-enhanced computed tomography [CT], magnetic resonance imaging [MRI], or transabdominal ultrasonography). Importantly, approximately 85% of patients with acute pancreatitis have acute interstitial edematous pancreatitis (answer choice C) characterized by an enlargement of the pancreas due to inflammatory edema. Due to the 3-hour onset of symptoms, serum lipase and amylase would be expected to be within the normal range. Serum lipase rises within 4-8 hours of the onset of symptoms, peaks at 24 hours, and returns to normal within 8-14 days.
Question:
A 198-cm (6 ft 6 in) tall, 18-year-old athlete suddenly passes out and collapses during a football game. He was unresponsive and numerous attempts to resuscitate him were unsuccessful. One month prior, physical examination showed a fourth heart sound, a systolic ejection murmur, and an arm span that was greater than his height. ECG showed prominent abnormal Q waves in leads II, III, and aVF. Which of the following is the most likely diagnosis?
- Abdominal aortic aneurysm rupture
- Aortic aneurysm rupture
- Aortic regurgitation
- Aortic stenosis
- Hereditary hypertrophic cardiomyopathy
- Mitral regurgitation
- Mitral stenosis
- Mitral valve prolapse
- Patent foramen ovale
- Pulmonary embolism
- Spontaneous pneumothorax
- Subaortic stenosis
- Traumatic pneumothorax
- Ventricular wall rupture
- Volume depletion
Answer: E – Hereditary hypertrophic cardiomyopathy. HCM is characterized by left ventricular hypertrophy (LVH) with a wide array of clinical manifestations. The physical examination in a patient with HCM may be normal or may reveal nonspecific abnormalities such as a fourth heart sound, systolic murmur, and/or a left ventricular lift. Prominent abnormal Q waves, particularly in the inferior (II, III, and aVF) and lateral leads (I, aVL, and V4-V6). These changes reflect septal depolarization of the hypertrophied myopathic tissue.
Question:
A 53-year-old man comes to the physician for a history and physical. Physical examination is unremarkable. Laboratory studies show total serum cholesterol of 280 mg/dL, LDL 165 mg/dL, HDL 40 mg/dL, and triglycerides 200 mg/dL. He is started on a treatment. What is the most common side effect associated with this drug?
- Agranulocytosis
- Angioedema
- Cataracts
- Cough
- Heart failure
- Hepatic failure
- Hypersensitivity reaction
- Lupus
- Myopathy
- Neuropathy
- Ophthalmoplegia
- Optic neuritis
- Pancreatitis
- Pulmonary fibrosis
- Rash
- Renal failure
- Respiratory failure
- Tinnitus
Answer: I - Myopathy. More lipophilic HMG-CoA reductase inhibitors (e.g. simvastatin, lovastatin, or atorvastatin) may be associated with more adverse events compared to hydrophilic statins (e.g. pravastatin and rosuvastatin). Hepatic dysfunction has been a source of concern; however, the actual risk appears to be very small. Myopathy remains an important side effect and is considered the most common side effect for the treatment of hypercholesterolemia from lipophilic HMG-CoA reductase inhibitors.
Question:
A 20-year-old IVDA comes to the emergency department with a 6-hour history left lower abdominal pain, nausea, and vomiting. He now states that the pain has shifted to the right lower region. Physical examination shows a temperature of 38.3°C (101°F). Laboratory tests show leukocytosis. Which of the following is the best next step of management?
- Abdominal CT
- Abdominal ultrasound
- Abdominal x-ray
- Initiate empiric antibiotics
- Observation
- Proceed to the operating room
- Psychiatry consult for malingering
- Psychiatry consult for factitious disorder
- Repeat laboratory testing
Answer: F – Proceed to the operating room. This patient is presenting with acute appendicitis (e.g. RLQ abdominal pain, nausea/vomiting, fever, leukocytosis, etc.). Patients in whom appendicitis is considered to be extremely likely after assessment, should proceed directly to the OR for an appendectomy without further radiologic or laboratory testing. Unnecessary delays may result in perforation or rupture of the appendix. |